what number is not a common denominator of 2/4 and 3/8

Answers

Answer 1
To find the LCM, write multiples of the three denominators:
2
:
2
,
4
,
6
,
8
,
10
,
12
,
...

4
:
4
,
8
,
12
,
16
,
20
,
...

8
:
8
,
16
,
24
,
32
,
...

The LCM is the smallest multiple that is common to all three:
8

This means the least common denominator is
8

Related Questions

Write the equation of the line in fully simplified slope intercept form

Answers

Answer:

y = 3x - 8

Step-by-step explanation:

m = slope

slope = y2 - y1 / x2 - x1

4 - 1 / 4 - 3 = 3/1 = 3

m = 3

b = y-intercept

b = -8

y = mx + b

y = 3x + (-8)

y = 3x - 8

Evaluate the expression when b= -4 and x= 2. b-7x​

Answers

Answer:

Step-by-step explanation:

b  - 7x = (-4) - 7*2

          = -4 - 14      {both numbers have same sign, so add and put the - sign for the result}

         = -18

the sum of two numbers is 22And their product is 105 what are the numbers?

Answers

Answer:

15 and 7 Im almost certain

Step-by-step explanation:

Jimmy and Ted each improved their yards by planting daylilies and shrubs. They brought theurbsupplies from the same store. Jimmy spent $80 on 4 daylilies and 9 shrubs. Ted spent $148 on daylillies and 4 of shrubs. What is the cost of one daylily and the cost of one shrub?​

Answers

Step-by-step explanation:

given,

JIMMY bought,

no. of daylillies = 4

no. of shrubs = 9

total cost of all material he bought = $80

TED bought,

no. of daylillies = 12

no. of shrubs = 4

total cost of all material he bought = $148

let daylillies be x and shrubs be y

the equation is,

4x + 9y = $80 -----------(1)

12x + 4y = $148 -----------(2)

now, multiply (1) by 3 we got,

12x + 27y = $240 -----------(3)

now, subtract (2) from (3) we got,

23y = 92

y = 92/23 = 4

y = 4

therefore cost of shrubs are $4.

so, cost of daylillies,

12x + 4(4) = 148

12x = 148 - 16 = 132

x = 132/12 = 11

x = 11

Hence, cost of daylillies are $11 and shrubs are $4.

hope this answer helps you dear......take care and may u have a great day ahead!

help!
[tex]8r+9-4r-3r-8r[/tex]
Thanks!

Answers

[tex]\\ \sf\longmapsto 8r+9-4r-3r-8r[/tex]

[tex]\\ \sf\longmapsto 8r-4r-3r-8r+9[/tex]

[tex]\\ \sf\longmapsto -7r+9[/tex]

[tex]\\ \sf\longmapsto 9-7r[/tex]

[tex]▪▪▪▪▪▪▪▪▪▪▪▪▪ {\huge\mathfrak{Answer}}▪▪▪▪▪▪▪▪▪▪▪▪▪▪[/tex]

Here's the solution for You !

just combine the like terms :

[tex]8r + 9 - 4r - 3r - 8r[/tex]

[tex] - 7r + 9[/tex]

what is the simplified form of 6^square root of x multiplied by 6^square root of x multiplied by 6^square root of x multiplied by 6^square root of x?
1. x^2/3
2. x^3/2
3. 4^6 square root of x
4. ^6 square root of 4x

Answers

Answer:

x^2/3, this the correct answer.

Step-by-step explanation:

It's simple!

=x^1/6+1/6+1/6+1/6

=x^4/6

=x^2/3

CAN SOMEONE HELP ME PLZZZ!!!!!!

Answers

1. figure 4 would be 81

Explanation:

from figure 1 wich is 3, if you keep multiplying 3 until figure 4. the shaded triangle would be 81

1x3 - 3

3x3 - 9

9x3 - 27

27x3 - 81

2. figure 10 would be 59,049

Explanation:

from figure 1 wich is 3, if you keep multiplying 3 until figure 10. the shaded triangle would be 59,049

1x3 - 3

3x3 - 9

9x3 - 27

27x3 - 81

81x3 - 243

243x3 - 729

729x3 - 2,187

2,187x3 - 6,561

6,561x3 - 19,683

19,683x3 - 59,049

3. it would be figure 8 wich is 6,561

Explanation:

1x3 - 3

3x3 - 9

9x3 - 27

27x3 - 81

81x3 - 243

243x3 - 729

729x3 - 2,187

2,187x3 - 6,561

what is the answer to 6=a/4+2

Answers

Answer:

a = 36

Step-by-step explanation:

6 = [tex]\frac{a}{4+2}[/tex] , that is

6 = [tex]\frac{a}{6}[/tex] ( multiply both sides by 6 to clear the fraction )

36 = a

Is the triangle proof correct? Photo attacted

Answers

Answer:

is good

Step-by-step explanation:

its algebra I need help

Answers

Answer:

B) 6

Step-by-step explanation:

4^2 = 16

16 - (2+8)

16 - 10

6

Hope this helps! Pls give brainliest!

Use the figure at the right. If BE = 3x + 6 and DF = 7x +4, find DF.

Answers

Line segments can be divided into sub-segments.

The value of DF is 18

The given parameters are:

[tex]\mathbf{BE = 3x + 6}[/tex]

[tex]\mathbf{DF = 7x + 4}[/tex]

We have:

[tex]\mathbf{BE + DF = 30}[/tex]

So, we have:

[tex]\mathbf{3x + 6 + 7x + 4 = 30}[/tex]

[tex]\mathbf{10x + 10 = 30}[/tex]

Subtract 10 from both sides

[tex]\mathbf{10x = 20}[/tex]

Divide both sides by 10

[tex]\mathbf{x = 2}[/tex]

Substitute [tex]\mathbf{x = 2}[/tex] in [tex]\mathbf{DF = 7x + 4}[/tex]

[tex]\mathbf{DF = 7 \time 2 + 4}[/tex]

[tex]\mathbf{DF = 14 + 4}[/tex]

[tex]\mathbf{DF = 18}[/tex]

Hence, the value of DF is 18

Read more about line segments at:

https://brainly.com/question/17313387

HELP ASAP!!!!!!!!!!! YOU WILL GET POINTS FOR DOING THIS CORRECTLY

Seven 6th graders count their nickels and quarters. The chart shows information about how much money each 6th grader has. Fill in the missing information

Answers

Answer:

Hope this helps

Step-by-step explanation:

x/-4 + 14>20

someone help a girl out

Answers

Answer:

x < -20

Step-by-step explanation:

x / -4  + 14 >20

Subtract 14 from each side

x / -4 +14-14 > 20 -14

x / -4 > 6

Multiply each side by -4, remembering to flip the inequality

x / -4 * -4 < 6 * -4

x < -20

the first row the theater has 15 seats in it each subsequent row has 3 more seats that seats that the previous ow . if the last row has 78 sears, how many rows are in the theater a 21 b 78 c 77 d 45 e 22

Answers

Answer:

21

Sorry if not correct

But I think this is the answer

The number of rows in the considered theater which has 3 more seats that seats that the previous row, with first row having 15 and last row having 78 seats is given by: Option D: 22

What is arithmetic sequence?

An arithmetic sequence is sequence of integers with its adjacent terms differing with one common difference.

If the initial term of a sequence is 'a' and the common difference is of 'd', then we have the arithmetic sequence as:

[tex]a, a + d, a + 2d, ... , a + (n+1)d, ...[/tex]

Its nth term is

[tex]T_n = a + (n-1)d[/tex]

(for all positive integer values of n)

And thus, the common difference is

[tex]T_{n+1} - T_n[/tex]

for all positive integer values of n

For this case, we can use arithmetic sequence as the number of seats in each next row is 3 more than previous row, so there is constant difference.

Now, the first number is 15, so we have: a = 15,d = 3 (increasing, so positive).

Let there are 'n' row in the theater.

Then the nth term of the arithmetic sequence having a = 15 and d = 3 should be 78 (as the nth row from first to last is the last row, having 78 seats).

Thus, we get:

[tex]T_n = a + (n-1)d\\78 = 15 + (n-1)3\\78-15 =3n - 3\\\\n = \dfrac{63+3}{3} = 22[/tex]

Thus, the number of rows in the considered theater which has 3 more seats that seats that the previous row, with first row having 15 and last row having 78 seats is given by: Option D: 22

Learn more about arithmetic sequence here:

https://brainly.com/question/3702506

3(x + 2) = 2x + 5 , equation/solution

Answers

Answer:

x = - 1

Step-by-step explanation:

3(x + 2) = 2x + 5 ← distribute parenthesis on left side by 3

3x + 6 = 2x + 5 ( subtract 2x from both sides )

x + 6 = 5 ( subtract 6 from both sides )

x = - 1

Which quantity is proportional to 90⁄2?
Check all that are true.

45⁄1

180⁄4

270⁄8

15⁄3

180⁄3

Answers

45/1 and 180/4 Your Welcome

A uniform probability distribution is a continuous probability distribution where the probability that the random variable assumes a value in any interval of equal length is _____. a. different for each interval b. the same for each interval c. either different or the same depending on the magnitude of the standard deviation d. None of the answers is correct.

Answers

Uniform probability distribution describes a distribution which is continous and the probability that a random variable assumes a value in any interval of equal length is the same for each interval.

The likelihood that a randomly selected variable within intervals with equal length is equally likely in a uniform probability distribution.

An example is a deck of cards, hearts, spade, diamond and club are of equal length. Hence, the probability of randomly drawing a card from any of these intervals is the same (1/4).

Therefore, the true statement is the option B.

Learn more :https://brainly.com/question/18405415

31 billion scientific notation???

Answers

Answer: 3.1 x 10¹⁰

Explanation: None

When Emmy went to bed, it was a frigid 9°F outside. The temperature has been dropping 2°F each hour.
Write an equation that shows the relationship between the number of hours since Emmy went to bed, x, and the temperature in °F, y.

Answers

Answer:

y=9-2x

Step-by-step explanation:

The equation that shows the relationship between the number of hours since Emmy went to bed is y = -2x + 9.

A linear equation is given by:

y = mx + b;

m is the rate of change, b is the initial value of y and y,x are variables.

Let x represent the number of hours since Emmy went to bed and y the temperature in °F. Since temperature has been dropping 2°F each hour, m = -2. Also b = 9°F, Hence:

y = -2x + 9

The equation that shows the relationship between the number of hours since Emmy went to bed is y = -2x + 9.

Find out more at: https://brainly.com/question/13911928

what is 42/7 as a whole numder

Answers

The answer is 6 hole

Answer:

6

Step-by-step explanation:

Because 42/7 simplified is 6/1 and 6/1 = 6.

Also / means division, you can just divide 42 by 7 = 6.

:)

Lines q, r, s, and t are distinct lines in the same
plane. Decide if lines q and t
are parallel, perpendicular, or if there is not
enough information given.
q Ir, ris,
ris, sit
A. parallel
B. perpendicular
C. not enough information

Answers

I think it’s b cause I’ve done this before and I think I can remember that answer

Gabrielle’s age is two times Michaels age. The sum of their ages is 39. What is Mikhails age?

Answers

Based on the given information, Michael's age is 13, and Gabrielle's age is 26.

Let's represent Gabrielle's age as G and Michael's age as M.

Given that Gabrielle's age is two times Michael's age, we can express this relationship as:

G = 2M.

We also know that the sum of their ages is 39:

G + M = 39.

Now, we can substitute the value of G from the first equation into the second equation:

2M + M = 39.

Combining like terms:

3M = 39.

To isolate M, we divide both sides of the equation by 3:

M = 39 / 3.

Simplifying further:

M = 13.

Therefore, Michael's age is 13.

To confirm this result, we can substitute M = 13 back into the equation G = 2M:

G = 2(13).

G = 26.

So, Gabrielle's age is 26.

For similar question on age.

https://brainly.com/question/30994122  

#SPJ8

27.6 inches of rain fell over a 100 day period. What was the average amount of rain that fell per day? Enter a numerical answer only.

Answers

Answer:

Step-by-step explanation:

Hey ! I beleive this is the answer: 27.5/100=.275

Please help determining angle measures for my geometry class

Answers

Answer:

a, b, d, e, f

Step-by-step explanation:

only c is incorrect

help me math homework

Answers

Answer:

please keep the question clearly

Please help me solve this problem

Answers

The answer is x= - 23/2. Pls mark brainliest pls

Answer:

11) 1.612) -11 1/2

Step-by-step explanation:

[tex]11. \: \: - 2.5(4x - 4) = - 6 \\ - 2.5(4x) - 2.5(4) = - 6 \\ - 10x + 10 = - 6 \\ - 10x + 10 - 10 = - 6 - 10 \\ - 10x = - 16 \\ \frac{ - 10x}{ - 10} = \frac{ - 16}{ - 10} \\ x = 1.6 \\ [/tex]

[tex] \frac{1}{6} (x - 5) = \frac{1}{2} (x + 6) \\ \frac{1}{6} \times x + \frac{1}{6} ( - 5) = \frac{1}{2} \times x + \frac{1}{2} \times 6 \\ \frac{x}{6} - \frac{5}{6} = \frac{x}{2} + \frac{6}{2} \\ \frac{x - 5}{6} = \frac{x + 6}{2} \\ 2(x - 5) = 6(x + 6) \\ 2x - 10 = 6x + 36 \\ - 10 - 36 = 6x - 2x \\ - 46 = 4x \\ \frac{ - 46}{4} = \frac{4x}{4} \\ \frac{ - 23}{2} = x \\ - 11 \frac{1}{2} = x[/tex]

PLS HELP GIVING BRAINLY FAST

Tell whether if the following is a solution to the linear equation or not.
1. x = 3; y = -1 -5x + 2y = -17
2. (0, 6) - x + 2y = 10
3. (-5, -2) -5x + 2y = -21
4. x = -2; y = 5 -3x – 4y = -14
5. x = 4; y = 3 2x + 3y = 17

Answers

Answer:

1) Linear equation

2) Non Linear equation

3) Non Linear equation

4) Linear equation

5) Non Linear equation

i hope this helps!

Step-by-step explanation:

find (f-g) (x) if f(x)=-2x+4 and g(x)=x^2-3x+5

Answers

Answer:

−[tex]x^{2}[/tex] + [tex]x^[/tex] − 1

Answer:x=2

Step-by-step explanation:

hope it helps

Angles A and B are supplementary. If mA = 67, find mB.

Answers

Answer:

Angle A +Angle B=180

67+B=180

B=113°

George worked 30 hours
He was paid a bonus of £20

Work out George's total pay.

Harry worked 34 hours his total pay was £300

Work out the bonus Harry was paid.

Answers

Step-by-step explanation:

Work out George's total pay:

=total pay=(8x30)+20

=240+20=£260

Work out the bonus Harry was paid:

=Total pay=(8x34)+bonus

300-272+bonus

300/-272 -272/-272 +bonus

bonus=£28